Difference between revisions of "1985 AHSME Problems/Problem 26"

m (Problem)
(Fixed answer choices and improved formatting)
 
(One intermediate revision by one other user not shown)
Line 1: Line 1:
 
==Problem==
 
==Problem==
Find the least [[positive integer]] <math> n </math> for which <math> \frac{n-13}{5n+6} </math> is a non-zero reducible fraction.
+
Find the least positive integer <math>n</math> for which <math>\frac{n-13}{5n+6}</math> is a non-zero reducible fraction.
  
<math> \mathrm{(A)\ } 45 \qquad \mathrm{(B) \ }68 \qquad \mathrm{(C) \  } 155 \qquad \mathrm{(D) \  } 226 \qquad \mathrm{(E) \  }\text84 </math>
+
<math> \mathrm{(A)\ } 45 \qquad \mathrm{(B) \ }68 \qquad \mathrm{(C) \  } 155 \qquad \mathrm{(D) \  } 226 \qquad \mathrm{(E) \  }\text{none of these} </math>
  
 
==Solution==
 
==Solution==
For the fraction to be reducible, the greatest common factor of the numerator and the denominator must be greater than <math> 1 </math>. By the [[Euclidean algorithm]],
+
For the fraction to be reducible, the greatest common factor of the numerator and the denominator must be greater than <math>1</math>. Using the [[Euclidean algorithm]], we compute <cmath>\begin{align*}\gcd\left(5n+6,n-13\right) &= \gcd\left(5n+6-5(n-13),n-13\right) \\ &= \gcd\left(71,n-13\right).\end{align*}</cmath>
 
+
Since <math>71</math> is prime, it follows that this GCD will be <math>1</math> unless <math>(n-13)</math> is a multiple of <math>71</math>, which first occurs when <math>n = 71+13 = 84</math>, so the answer is <math>\boxed{\text{(E) none of these}}</math>.
<math> \gcd(5n+6, n-13) </math>
 
 
 
<math> \gcd(5n+6-5(n-13), n-13) </math>
 
 
 
<math> \gcd(71, n-13) </math>
 
 
 
Since <math> 71 </math> is prime, <math> n-13 </math> must be a multiple of <math> 71 </math>, which first occurs when <math> n=71+13=84, \boxed{\text{E}} </math>.
 
  
 
==See Also==
 
==See Also==
 
{{AHSME box|year=1985|num-b=25|num-a=27}}
 
{{AHSME box|year=1985|num-b=25|num-a=27}}
 
{{MAA Notice}}
 
{{MAA Notice}}

Latest revision as of 01:44, 20 March 2024

Problem

Find the least positive integer $n$ for which $\frac{n-13}{5n+6}$ is a non-zero reducible fraction.

$\mathrm{(A)\ } 45 \qquad \mathrm{(B) \ }68 \qquad \mathrm{(C) \  } 155 \qquad \mathrm{(D) \  } 226 \qquad \mathrm{(E) \  }\text{none of these}$

Solution

For the fraction to be reducible, the greatest common factor of the numerator and the denominator must be greater than $1$. Using the Euclidean algorithm, we compute \begin{align*}\gcd\left(5n+6,n-13\right) &= \gcd\left(5n+6-5(n-13),n-13\right) \\ &= \gcd\left(71,n-13\right).\end{align*} Since $71$ is prime, it follows that this GCD will be $1$ unless $(n-13)$ is a multiple of $71$, which first occurs when $n = 71+13 = 84$, so the answer is $\boxed{\text{(E) none of these}}$.

See Also

1985 AHSME (ProblemsAnswer KeyResources)
Preceded by
Problem 25
Followed by
Problem 27
1 2 3 4 5 6 7 8 9 10 11 12 13 14 15 16 17 18 19 20 21 22 23 24 25 26 27 28 29 30
All AHSME Problems and Solutions

The problems on this page are copyrighted by the Mathematical Association of America's American Mathematics Competitions. AMC logo.png